1050
$\begingroup$

The first thing to say is that this is not the same as the question about interesting mathematical mistakes. I am interested about the type of false beliefs that many intelligent people have while they are learning mathematics, but quickly abandon when their mistake is pointed out -- and also in why they have these beliefs. So in a sense I am interested in commonplace mathematical mistakes.

Let me give a couple of examples to show the kind of thing I mean. When teaching complex analysis, I often come across people who do not realize that they have four incompatible beliefs in their heads simultaneously. These are

(i) a bounded entire function is constant;
(ii) $\sin z$ is a bounded function;
(iii) $\sin z$ is defined and analytic everywhere on $\mathbb{C}$;
(iv) $\sin z$ is not a constant function.

Obviously, it is (ii) that is false. I think probably many people visualize the extension of $\sin z$ to the complex plane as a doubly periodic function, until someone points out that that is complete nonsense.

A second example is the statement that an open dense subset $U$ of $\mathbb{R}$ must be the whole of $\mathbb{R}$. The "proof" of this statement is that every point $x$ is arbitrarily close to a point $u$ in $U$, so when you put a small neighbourhood about $u$ it must contain $x$.

Since I'm asking for a good list of examples, and since it's more like a psychological question than a mathematical one, I think I'd better make it community wiki. The properties I'd most like from examples are that they are from reasonably advanced mathematics (so I'm less interested in very elementary false statements like $(x+y)^2=x^2+y^2$, even if they are widely believed) and that the reasons they are found plausible are quite varied.

$\endgroup$
31
  • 138
    $\begingroup$ I have to say this is proving to be one of the more useful CW big-list questions on the site... $\endgroup$ May 6, 2010 at 0:55
  • 37
    $\begingroup$ The answers below are truly informative. Big thanks for your question. I have always loved your post here in MO and wordpress. $\endgroup$
    – Unknown
    May 22, 2010 at 9:04
  • 32
    $\begingroup$ wouldn't it be great to compile all the nice examples (and some of the most relevant discussion / comments) presented below into a little writeup? that would make for a highly educative and entertaining read. $\endgroup$
    – Suvrit
    Sep 20, 2010 at 12:39
  • 28
    $\begingroup$ It's a thought -- I might consider it. $\endgroup$
    – gowers
    Oct 4, 2010 at 20:13
  • 29
    $\begingroup$ Meta created tea.mathoverflow.net/discussion/1165/… $\endgroup$
    – user9072
    Oct 8, 2011 at 14:27

292 Answers 292

1
6
7
8 9 10
10
$\begingroup$

Multiplication of differential forms is inherently anti-commutative. Thus, if $x$ and $y$ are coordinates on a surface, then $dx \wedge dy$ makes sense but $(dx)^2+(dy)^2$ is either nonsense or, if it means anything, is $0$.

I'm not sure why I believed this, but I did for several years. I tried my best to avoid creating this impression in my students, but I think it still happened in some of them, simply because the curriculum spends a lot of time on integration and Stokes theorem and very little time on metrics, curvature, etc.

$\endgroup$
1
  • 1
    $\begingroup$ This is about notations. You do the multiplication of differential forms in exterior algebra, but for metrics, you do it in tensor algebra. By the way, the textbooks that I read don't write dx², but write a symmetric bilinear form explicitly. $\endgroup$
    – user20948
    Feb 8, 2021 at 22:05
10
$\begingroup$

Probably my fault for not paying enough attention in analysis, but:

Any continuous function on the interval that has derivative equal to zero almost everywhere is constant.

$\endgroup$
10
$\begingroup$

A polynomial $p(x)$ of degree $n$, with coefficients in a commutative ring $R$, has at most $n$ roots, counting multiplicity. This is true if $R$ is an integral domain, but it can fail in the presence of zero divisors.

For instance, $p(x) = x^2+5x$ has four solutions when $R = \mathbb{Z}/6\mathbb{Z}$. I realized this mistake when a colleague asked me about factorization over a non-commutative ring, and I realized that I did not even know what would happen in the presence of zero-divisors.

This does motivate a question I have not found an answer to: is the number of solutions of $p(x)$ bounded by a function of the degree and the characteristic of the ring?

$\endgroup$
4
  • 7
    $\begingroup$ To answer your last question, $x^2-1$ has an infinite number of roots in $k^{\mathbb{N}}$ for any nonzero ring $k$. So, no. $\endgroup$
    – Gro-Tsen
    Apr 20, 2016 at 17:00
  • 2
    $\begingroup$ Doesn't it only have one solution in $k^\mathbb{N}$ if $k$ has characteristic 2, @Gro-Tsen? $\endgroup$ Apr 20, 2016 at 17:36
  • 2
    $\begingroup$ @OmarAntolín-Camarena Oh right, what I wanted to write was $x^2-x$, and I got confused between "idempotent" and "one-potent"(?). But of course $x^2-1$ also works provided, as you point out, that $1\neq -1$ in $k$. $\endgroup$
    – Gro-Tsen
    Apr 20, 2016 at 20:31
  • 1
    $\begingroup$ A similar eample is $\mathbb Z/4 \mathbb Z[t]$. For each $n$ the element $2t^n$ is a root of $x^2=0$. $\endgroup$
    – Nick S
    Mar 3, 2020 at 6:17
10
$\begingroup$

Here is a short list of some false beliefs I had when I was studying mathematics, I suppose they may be common but I have never checked:

  • I was in the last year of high school and studying university-level math in advance. I remember trying for a week to prove that a continuous injective map from an open subset of $\mathbb{R}^2$ to $\mathbb{R}^2$ that preserves "being aligned" (I mean that maps aligned triples to aligned triples) must be the restriction of an affine map (over $\mathbb{R}$). That is disproved by restrictions of projective transformations... Which I knew of but I was not able to see they contradicted my belief. When my teacher told me "What about projective transformations?"... I felt dumb.
  • I was in the 1st year of PhD studies. My advisor, Adrien Douady, had an idea to build polynomial Julia sets with positive Lebesgue measure. Julia sets are fractals, often with complicated topological structures at every scale. Surely that must be the source of measure? So as an exercise, I tried for a week to prove that Jordan curves are necessarily of Lebesgue measure 0. I told Adrien about my attempts. He gave me a counter-example. I felt dumb.
  • Learning that there are closed subsets of the interval with positive Lebesgue measure but no interior did not surprise me as much, as the construction is very simple, but still that's a bit counterintuitive.
  • When you zoom on the Mandelbrot set, you see all that round components with smooth boundaries. They look so round. Surely they must be circles, for otherwise the difference would be visible. Well... they are not (except one). Guess how I felt when I learned.
  • Frankly, when learning the first time about complex numbers, did anybody here expect that, adding the square root of -1 to the reals would add the roots of all other polynomials?
  • I was giving a lecture to math teachers about sensitivity to initial condition (call it chaos) and showing strange attractors on the computer, one told me that by the very presence of chaos, what we see may be quite far from the actual behaviour of the equation, save reality. It turns out hyperbolic systems are stable, so I believe this is still representative (it does not prove it but it is an encouraging hint).
  • ... Chaos in deterministic systems. I won't develop on that.
  • Surely before hearing of set theory and Cantor's argument, you will believe that all sets are countable. Then after learning that this is not the case, you will think that $\mathbb{R}^2$ must be bigger than $\mathbb{R}$, right?
  • You have a $C^\infty$ function on the right half plane, all of whose derivatives have a continuous extension to the boundary line. Surely, it must be easy to extend it to a $C^\infty$ function of the whole plane, isn't it? Well... You can but I would not call it easy.
  • Short statements have short proofs. Disproved by Fermat's last theorem (among others).
  • I was quite disappointed to learn that there cannot be a finite non-commutative field (division algebra).

I have a few other examples, that I would not term "common false beliefs" but rather "fun and surprising math facts". Is there already a MO question about that?

$\endgroup$
10
$\begingroup$

Here's another howler some people commit: If $m$, $n$ are integers such that $m$ divides $n^2$ then $m$ divides $n$.

It's true sometimes, for example if $m$ is prime (or more generally squarefree, i.e. a product of distinct primes). But in general all one can conclude is that there exists integers $p$, $q$, $r$ with $p$ squarefree such that $ m = p q^2 $ and $ n = p q r $

The usual counterexample is that $8$ divides $4^2$ but not $4$ ;-)

$\endgroup$
1
  • 17
    $\begingroup$ An even more trivial counterexample is that 4 divides 2^2 but not 2 :-P $\endgroup$ Feb 23, 2011 at 9:40
10
$\begingroup$

For some reason, I believed until recently that every compact operator on a Hilbert space must have eigenvalues. It's obviously true for finite-rank operators and Hermitian operators, so I must have subconsciously generalized.

This was despite being well aware of (say) the Volterra operator, say; I simply had two contradictory ideas in my head.

$\endgroup$
2
  • $\begingroup$ The example I had heard of for this was the operator on $\ell^2$ you get by multiplying pointwise by $\frac{1}{n}$ after doing a unilateral right shift. I realize now that this is the Volterra operator in disguise. $\endgroup$ Jun 2, 2021 at 2:31
  • 1
    $\begingroup$ Yes, your example was what I first thought of as well, and only afterwards did someone bring up the Volterra operator. $\endgroup$ Jun 2, 2021 at 19:30
9
$\begingroup$

"The set A = {a, b} has two elements..."

It's quite simple to notice that a can be the same as b, but after 5 years of university there were people still believing it...

$\endgroup$
8
  • 22
    $\begingroup$ I'm not sure there is a false belief here, as much as awkward writing. Depending on context, I might very well write "The set $\{a,b \}$ (where $a$ and $b$ might be equal)..." if this issue mattered. $\endgroup$ May 6, 2010 at 11:16
  • 1
    $\begingroup$ There are many situations where one needs to speak of a set of two numbers that may or may not be equal. E.g.: "Let x<sub>1</sub>, x<sub>2</sub> &isin; ℝ. Then among all the open intervals containing the set {x<sub>1</sub>, x<sub>2</sub>}, none of them is contained in all the others." If one is addressing mathematicians, there is no need to specify that x<sub>1</sub> might be equal to x<sub>2</sub>. $\endgroup$ Jun 17, 2010 at 23:34
  • 1
    $\begingroup$ E.g. if you said something like "for all a,b, (in some given universe) the set {a,b} has two elements", then I would agree. $\endgroup$
    – roy smith
    Dec 1, 2010 at 19:35
  • 2
    $\begingroup$ Single-letter symbols are usually assumed to be variables, if the context doesn't determine otherwise, even in the absence of quantifiers. (You can put in an implicit universal quantifier to close up all sentences.) $\endgroup$ Apr 4, 2011 at 9:41
  • 3
    $\begingroup$ Here is a related but slightly less obvious situation. The ordered pair $(a,b)$ is generally defined in set theory to be $\{\{a\},\{a,b\}\}$. This is generally thought of as a set with two elements. But what if $a=b$? $\endgroup$ Sep 26, 2014 at 4:12
9
$\begingroup$

I saw many students using the "fact" that for a subset $S$ of a group one has $SS^{-1}=\{e\}$

$\endgroup$
1
  • 6
    $\begingroup$ This is an interesting example, because it addresses the mistakes that come from the all-too frequent confusion with notations. But we need our shortcuts, our $f^{-1}(x)$ versus $x^{-1}$, etc. Obtaining concise notations while avoiding confusion: a tricky proposition! $\endgroup$ Apr 14, 2011 at 15:50
9
$\begingroup$

Taylor's Formula and displacement operator: I (too often) see in papers (mathematical physics but a recent paper (a) by mathematicians also) the statement

False belief 1 : a) Let $D=\frac{d}{dx}$ be the derivation operator. Then, for all $f\in C^\infty(\mathbb{R})$, $$ e^{tD}[f](x)=f(x+t) $$

which is false (take any $\phi\in C^\infty(\mathbb{R})$ with compact support, for instance).

False belief 2 : b) In the same vein, for formal power series (``our object is formal then we do not have to ensure convergences''). Let $S(x)\in \mathbb{R}[[x]]$ ($x$ is a formal variable) then for $t\in \mathbb{R}$, one has $$ e^{tD}[S](x)=S(x+t) $$

which is false as we must have $t$ in the domain of convergence of $S$.

Remarks (i) The function $f\in C^\infty(\mathbb{R})$ is analytic over $\mathbb{R}$ iff $$ (\forall x\in \mathbb{R})(\exists R>0)(\forall t\in ]-R,R[) (\sum_{n\geq 0}\frac{t^n}{n!}D^n[f](x)=f(x+t))\qquad (1) $$ (ii) Even if $f\in C^\omega(\mathbb{R})$, it can happen that the left hand side of eq. (1) do not converge otherwise $f$ would be the restriction of an entire function (which e.g. $\frac{1}{1+x^2}$ is not, for example).

(iii) Even if the LHS of (1) converges for all $x,t\in \mathbb{R}$, $f$ need not be analytic. Consider the following function (classic in theory of distributions)
$$ f(x)=0\mbox{ if } x\notin ]-1,1[\mbox{ and } f(x)=e^{\frac{1}{1-x^2}} \mbox{ if } x\in ]-1,1[ $$ (iv) In the (b) case $S=\sum_{n\geq 0}n!\, x^n$ for example cannot be displaced.

$\endgroup$
7
  • $\begingroup$ What is the correct statement? $\endgroup$
    – ಠ_ಠ
    Jul 13, 2017 at 7:33
  • $\begingroup$ @ಠ_ಠ: it is true for real analytic functions defined on the entire real number line. That is one possible choice of correct statement. $\endgroup$
    – Ben McKay
    Jul 13, 2017 at 8:07
  • 2
    $\begingroup$ Maybe you could point out the error in my reasoning: for a Lie group $G$, $G$ has a canonical left action on $C^\infty(G, \mathbb{R})$ by $(g.f)(x) =f(g^{-1}.x)$. Since $D=\frac{d}{dx}$ is a left-invariant vector field on $(\mathbb{R}, +)$, then $(e^{tD}.f) (x)=f(e^{-tD}.x)=f(x-t)$. $\endgroup$
    – ಠ_ಠ
    Jul 13, 2017 at 8:35
  • 1
    $\begingroup$ I think we probably mean different things with our notation. For me, $\exp: \mathfrak{g} \to G$ always denotes the exponential map of the Lie group, which always exists. But it looks to me like you mean by this notation that you are integrating the Lie algebra action rather than the Lie algebra. $\endgroup$
    – ಠ_ಠ
    Jul 13, 2017 at 21:02
  • 3
    $\begingroup$ @ಠ_ಠ I think I see what you meant. In the (Fréchet) space $C^\infty(\mathbb{R})$, the evolution equation $$ y(0)=f ; y′(t)=D[y(t)] $$ has a (unique) solution $y(t)[x]=f(x+t)$ which we could (legitimately ?) note $y(t)=e^{tD}[f]$, but one must keep in mind that, in this case $e^{tD}$ cannot be developed without caution. $\endgroup$ Jul 16, 2017 at 14:17
9
$\begingroup$

"This algebraic variety is a $C^\infty$-smooth manifold, therefore it must be non-singular". This sounds obvious (and in fact it is true over $\mathbb{C}$) however it is false in general (for instance over $\mathbb{R}$). See the discussion here for many details.

$\endgroup$
9
$\begingroup$

A somewhat common belief among students starting out in cryptography:

Breaking RSA requires factoring the modulus.

Although it is not quite known to be a "false" belief, there is no known reduction showing that breaking RSA implies finding the prime factors of the modulus. This in contrast to e.g. Rabin's cryptosystem, and various cryptographic schemes built on other hard problems, whose security provably relies on the underlying hard problems.

$\endgroup$
5
  • 2
    $\begingroup$ I would put this on the same level as the "false belief" that NP-complete problems really are hard. Nobody knows for sure if it actually is false or not, but given the current state of affairs it is a somewhat reasonable conjecture. And the very fact that nobody knows how to do it means that in practical applications NP-complete problems really are hard to solve and breaking RSA really does require factorisation of the modulus. $\endgroup$ Mar 16, 2018 at 22:56
  • 1
    $\begingroup$ Unless you say what the term "breaking RSA" is supposed to mean then it's impossible to evaluate whether that is a false belief or not. For example, say $m = pq$ is an RSA modulus with two different primes $p$ and $q$ and $e$ and $d$ are encryption and decryption exponents for that modulus (so $e$ an$d$ are positive integers such that $ed \equiv 1 \bmod \varphi(m)$). In practice $e > 1$ and $d > 1$, and also in practice $p$ and $q$ are both odd. If "Breaking RSA" means somehow determining $\varphi(m)$ from $m$ and $e$ then you can factor $m$ from knowing $\varphi(m)$ and $m$. If (cont.) ... $\endgroup$
    – KConrad
    Feb 25, 2019 at 11:33
  • $\begingroup$ ... "Breaking RSA" means somehow determining $d$ from $m$ and $e$, then you know $ed - 1$, which is a positive multiple of $\varphi(m)$, and there is a probabilistic algorithm that with over 50% probability of success each time will lead to a nontrivial factor of $m$ starting with a random choice of an integer from 1 to $m-1$. See Theorem 5.6 of kconrad.math.uconn.edu/blurbs/ugradnumthy/RSAnotes.pdf. If "Breaking RSA" means "a wizard tells you how to decode each message without explaining how it is done" then that's not math and thus can't be judged as being a false belief in math. $\endgroup$
    – KConrad
    Feb 25, 2019 at 11:36
  • 1
    $\begingroup$ @KConrad This answer is referring to the fact that, unlike for the mentioned Rabin variant, the RSA problem is not known to be as hard as factoring: given an oracle that decrypts ciphertexts, there is no efficient way known to use this oracle to recover the factorization of the modulus. This in contrast to many other schemes and protocols, for which such an oracle would immediately yield the solution for the hard problem it is presumed to rely on. (And many would disagree with you that such oracle reductions are "not math"...) $\endgroup$
    – TMM
    Apr 2, 2019 at 23:46
  • $\begingroup$ @JohannesHahn I'd say the difference is that in cryptography, there do exist plenty reductions for other schemes proving that breaking these schemes implies solving the underlying well-studied hard problems. The whole hardness-hierarchy at least gives us some faith that some problems are probably really hard, but the RSA decoding problem is not connected to factoring in this hierarchy, let alone to any NP-hard problems. $\endgroup$
    – TMM
    Apr 4, 2019 at 1:24
9
$\begingroup$

Heisenberg-Weyl and enveloping algebras

I have heard (and have read) the following belief even among distinguished mathematical physicists.

Let $HW_\mathbb{C}$ be the (associative with unit) algebra generated by two elements $\{a,a^\dagger\}$ subjected to the relation $[a,a^\dagger]=1$. It is easy to check that $$ \mathfrak{g}=span_\mathbb{C}\{a,a^\dagger,1_{HW_\mathbb{C}}\} $$ is a Lie subalgebra.

False belief The algebra $HW_\mathbb{C}$ is the universal enveloping algebra of $\mathfrak{g}$ i.e. $$ HW_\mathbb{C}=U(\mathfrak{g})\ . $$

One way to see that this is false at once is to observe that any enveloping algebra $U(\mathfrak{g})$ possesses (at least) a character $\varepsilon$, but there is none on $HW_\mathbb{C}$ (one would have indeed $1=\varepsilon([a,a^\dagger])=0$).

Late edit (After Darij's post) Indeed, the enveloping algebra of $$ \mathfrak{g}=span_\mathbb{C}\{a,a^\dagger,1_{HW_\mathbb{C}}\} $$ is the algebra (associative with unit) generated by $\{a,a^\dagger,e\}$ ($e$ is a clone of $1_{HW_\mathbb{C}}$) subjected to the relations $$ [a,a^\dagger]=e ; [a,e]=[a^\dagger,e]=0\qquad \mbox{[Rel 1]} $$ and the canonical arrow $U(\mathfrak{g})\to HW_\mathbb{C}$ transforms an expression in $a,a^\dagger,e$, already reduced by [Rel 1] into its image in $HW_\mathbb{C}$. For relations [Rel 1] one can take, for instance, the normal form: all $a^\dagger$ (creations) on the left, all $a$ (annihilations) on the right and $e$ anywhere (as they are central).

Combinatorially, in the reduced expressions of $U(\mathfrak{g})$, the power of e counts the number of times Wick commutations have been applied. I'd include this in my post if it were not out of place.

$\endgroup$
5
  • 3
    $\begingroup$ Yeah. There is a surjection $U\left(\mathfrak{g}\right) \to HW_{\mathbb{C}}$, though; its kernel is the ideal generated by $1_{HW_{\mathbb{C}}} - 1$. Thus, $HW_{\mathbb{C}}$ can be viewed as a "reduced" $U\left(\mathfrak{g}\right)$ (similarly to how the free product $A * B$ of two algebras $A$ and $B$ can be viewed as a "reduced" tensor algebra $T\left(A \otimes B\right)$). This is one of the things Pavel Etingof taught me; before that, I thought Weyl/Clifford algebras and universal enveloping algebras were similar but un-relatable constructions. $\endgroup$ Oct 3, 2017 at 6:34
  • $\begingroup$ I suppose a simple way to convince oneself this is not right is simply to note that the `unit' in $\mathfrak g$ is not special in any way (and in particular, does not become the unit of the universal envelope), as darij does. $\endgroup$
    – Pedro
    Mar 21, 2021 at 20:36
  • 1
    $\begingroup$ @PedroTamaroff Of course yes. Units have no meaning in the world of Lie algebras. Note that, beyond the (philosophical) question of the meaning, the canonical arrow $U(\mathfrak{g})\to HW_\mathbb{C}$ has a combinatorial interest as the reduction counts something. $\endgroup$ Mar 22, 2021 at 7:29
  • $\begingroup$ @darijgrinberg [$𝐴$ and $𝐵$ can be viewed as a "reduced" tensor algebra $𝑇(𝐴\otimes 𝐵)$]---> didn't you mean $𝑇(𝐴\oplus 𝐵)$]? $\endgroup$ Mar 22, 2021 at 8:05
  • $\begingroup$ Well, as a second thought, both are possible. The $T(A\oplus B)$ version allows noncommutative grading and embedding proofs though. $\endgroup$ Mar 22, 2021 at 8:23
9
$\begingroup$

"Frac" and "Const" commute: A differential ring is simply a pair $(R,\partial)$ where $R$ is a ring and $\partial$ is a derivation in $R$ (i.e. $\partial\in End_{\mathbb{Z}}(R)$ follows identically Liebniz rule $\partial(ab)=\partial(a)b+a\partial(b)$). The constants of $R$, $\ker(\partial)$ form a subring $Const(R)\subset R$ called subring of constants.

If $R$ is a commutative domain (i.e. without zero divisors), it is standard to consider the field of fractions $Frac(R)$ and to extend $\partial$ by the formula of calculus $\partial(1/g)=-\partial(g)/g^2$.

False belief: $Const(Frac(R))=Frac(Const(R))$, in other words, every constant in $Frac(R)$ is of the form $\alpha/\beta$, where $\alpha,\beta\in Const(R)$.

Until yesterday morning, I postponed to prove this (false) lemma (thinking it was an easy exercise). Then, preparing a talk, I could not prove this and searched for a counterexample. Finally I found a simple one in MSE (see below).

See there for a discussion and counterexamples.

$\endgroup$
9
$\begingroup$

A common misconception in analysis is that if two $C^1$-functions are linearly independent then their Wronskian is non-zero at some point.

This is the result of carelessly reversing a true implication. The mistake was first made in 1882 by Thomas Muir (who is responsible for the name "Wronskian") in his "Treatise on the Theory of Determinants" and Giuseppe Peano pointed out a counterexample as early as 1889 (it suffices to take $y=x^2$ and $y=x|x|$).

For more information on the history of this mistake see "Peano on Wronskians: A Translation". by Susannah M. Engdahl and Adam E. Parker.

$\endgroup$
8
$\begingroup$

Every matrix is the sum of a symmetric and an antisymmetric matrix. Hence:

If $V$ is a vector space and $k$ is a number, then the $k$-th tensor product of $V$ with itself decomposes as a direct sum into symmetric and antisymmetric tensors: $$ \underbrace{V \otimes ... \otimes V}_{k\text{ times}} = \Lambda^kV \oplus \mathrm{Sym}^kV $$

Recall (in the finite-dimensional case) the dimensions: $$ \dim \Lambda^k V = \binom{n}{k} \quad\text{ and }\quad \dim\mathrm{Sym}^kV = \binom{n+k-1}{k} $$

Looking at $k=1$ shows that we have non-trivial intersection.

Looking at $n=k=3$ shows that the sum is not exhausting.

$\endgroup$
2
  • 2
    $\begingroup$ Is this a common false belief? $\endgroup$
    – Jim Conant
    Oct 18, 2015 at 2:52
  • 2
    $\begingroup$ @JimConant: I believed this once. Of course, if you count dimensions it's obviously false. But if $V$ is an infinite-dimensional Hilbert space it sure seems natural to decompose the full tensor product into its Bosonic and Fermionic parts, and you might not think right away to ask whether it works in finite dimensions. That's my excuse, anyway! $\endgroup$
    – Nik Weaver
    Mar 4, 2016 at 4:22
8
$\begingroup$

Maybe I am a bit late to the party, but here is something that I falsely believed in for a while:

False Belief: If $f$ is a continuously differentiable function with a horizontal asymptote, i.e. $\lim_{x\to +\infty}f(x)=L$, then $f'(x)\to 0$ as $x$ goes to infinity.

Of course, this is incorrect even for smooth functions; take $g$ to be a smooth $L^1$ function with oscillatory behavior so that $\lim_{x\to +\infty}g(x)$ does not exist and then set $f(x)=\int_{0}^xg(s)ds$.

I came to this misconception during an ODE course I took in my undergrad. In the study of the asymptotic behavior of solutions of an autonomous equation $$ \frac{dy}{dx}=F(y) $$ one argues that if a solution $y=y(x)$ tends to a limit $L$ as $x\to +\infty$, that limit must be a zero of the vector field. We then proceeds with determining which zeros of $F$ are asymptotically stable to find $L$. The point is that here not only $\lim_{x\to +\infty}y$, but also $\lim_{x\to +\infty}\frac{dy}{dx}$ exists due to the ODE. Hence:

Correct Statement: If $f$ is a continuously differentiable function with a horizontal asymptote, and if $f'$ admits a limit as $x$ goes to infinity, that limit must be zero.

This readily follows from the mean value theorem.

$\endgroup$
1
  • 2
    $\begingroup$ A related old answer. I have found that people usually need some prompting to think of an $L^1$ function that does not converge to $0$ at infinity, even though all one needs is a sequence of spikes of increasing height, but sufficiently rapidly decreasing width so that the sum of the areas under them converges. $\endgroup$ Mar 3, 2020 at 17:02
8
$\begingroup$

It’s tempting to believe that a knot and its mirror-image, if tied in succession on a rope, can cancel. View https://youtu.be/lwWeRMmXIoU to see John Conway’s lucid explanation of why this is false.

I would be interested in knowing whether Lord Kelvin was under this misapprehension when he proposed his vortex theory of the atom. (Knots don’t cancel but vortices can.)

I once went to a conference on recreational mathematics in which a speaker claimed that a knot and its mirror-image can be cancelled; I was kind enough not to ask for a demonstration. (There is a way to pretend to make them cancel, but I don’t know the details of the trick.)

$\endgroup$
1
  • 2
    $\begingroup$ Although a knot and its mirror image do cancel in the concordance group! $\endgroup$
    – Jim Conant
    Jan 25, 2021 at 18:56
8
$\begingroup$

Maybe that's a cute one:

If $f$ is a continuous real function, then $f^{-1}(x)$ is at most countable unless $f$ is constant on some interval.

$\endgroup$
5
  • 5
    $\begingroup$ How would you stumble upon this belief? The most basic example of continuous functions (constant ones) fail it, so it seems like an easy belief to correct. $\endgroup$ Nov 14, 2020 at 21:00
  • $\begingroup$ Well, thanks, clarified what was left as obvious. $\endgroup$ Nov 14, 2020 at 21:04
  • 2
    $\begingroup$ Hmm, with the clarification added, really not obvious. What is an example of a counterexample? $\endgroup$
    – JoshuaZ
    Nov 14, 2020 at 23:21
  • 1
    $\begingroup$ Any component of a Peano curve does the job. $\endgroup$ Nov 14, 2020 at 23:59
  • 2
    $\begingroup$ You don't need that fancy Peano curve. Every closed set in a metric space is the zero set of a real valued function. $\endgroup$
    – user20948
    Feb 8, 2021 at 22:43
8
$\begingroup$

The "curse of dimensionality" means that in a hypercube the volume is increasingly concentrated in the corners as the number of dimensions increase.


In fact half the volume of a hypercube is closer to the centre than to the nearest vertex, with any number of dimensions.

The real curse is that the vast majority of the points of a unit hypercube of dimension $n$ are a distance less than $\frac{5}{n}$ from the outside of the hypercube, distances $\sqrt{\frac{n}{12}} \pm \frac12$ both from the centre and from the nearest vertex, and a distance $\sqrt{\frac{n}{6}} \pm 1$ from the vast majority of other points, which for large $n$ are narrow bands.

$\endgroup$
8
$\begingroup$

Surprisingly, I didn't find this answer in the ten pages above, so...

If two measures agree on a class generating their sigma-algebras, they are equal, right?

(And there actually are two ways in which this is false. Even knowing the total measure and assuming that it is finite is not enough to make this true)

$\endgroup$
10
  • 3
    $\begingroup$ If you don’t ask that the total measure is given, this fails already in a space with exactly two points. $\endgroup$ Dec 28, 2022 at 23:11
  • 2
    $\begingroup$ Precisely, so maybe you can consider dodging the trivial counterexample by adding these assumptions. $\endgroup$ Dec 28, 2022 at 23:26
  • 3
    $\begingroup$ @OfirGorodetsky, Consider the sigma-algebra of all subsets of $S = \{ 1, 2, 3, 4 \}$. Define $\mu \{ 1 \} = \mu \{ 2 \} = \mu \{ 3 \} = \mu \{ 4 \}$, $\nu \{ 1 \} = \nu \{ 3 \} = 0$ and $\nu \{ 2 \} = \nu \{ 4 \} = 2$. Then the total measures are equal: $\mu \{ 1, 2, 3, 4 \} = 4 = \nu \{ 1, 2, 3, 4 \}$ and $\mu$ and $\nu$ agree on $A = \{ \{ 1, 2 \}, \{ 1, 4 \} \}$ since both sets have measures 2. Finally, $2^S = \sigma ( A )$. $\endgroup$
    – tsnao
    Dec 29, 2022 at 20:40
  • 2
    $\begingroup$ To make the statement correct, one should assume that the generating set is closed under intersection (that is, it is a $\pi$-system) $\endgroup$
    – tsnao
    Dec 29, 2022 at 20:42
  • 2
    $\begingroup$ I forgot $\mu \{ 1 \} = 1$ above. $\endgroup$
    – tsnao
    Dec 29, 2022 at 20:52
8
$\begingroup$

I just learned incidentally in the comments of another question here that it is not true that every proper subgroup is contained in a maximal (proper) subgroup. A counterexample is easy to find: the additive group $\mathbb{Z}[\frac{1}{2}]$ has no maximal proper subgroup. But the confusion with proper ideals which are contained in a maximal (proper) ideal, by Zorn's lemma, is certainly something that will surprise others. (The reason for the difference, as I see it, is that in the case of ideals we can detect properness by $1 \not\in I$, whereas for subgroups there is no such criterion.)

$\endgroup$
3
  • $\begingroup$ Isn't $\mathbb{Z}$ a maximal proper subgroup of $\mathbb{Z}[\frac{1}{2}]$? $\endgroup$
    – Weier
    Dec 2, 2023 at 12:00
  • 1
    $\begingroup$ Consider the cyclic subgroup generated by $1/2$. It is proper in $\mathbb{Z}[\frac{1}{2}]$, and it properly contains $\mathbb{Z}$, so $\mathbb{Z}$ isn't maximal. $\endgroup$ Dec 2, 2023 at 12:14
  • $\begingroup$ Nevermind, I thought of $\frac{1}{2} \mathbb{Z}$ when reading $\mathbb{Z}[\frac{1}{2}]$... $\endgroup$
    – Weier
    Dec 2, 2023 at 12:19
7
$\begingroup$

Many people believe that Cantor proved the uncountability of the real line using a diagonal argument. This paper does not that provide that proof; Cantor's stated purpose was to prove the existence of `uncountable infinities' without using the theory of irrational numbers.

$\endgroup$
4
  • 2
    $\begingroup$ More to the point, I think, is that the paper proves that the power set of any set has greater cardinality than the set itself. This is the first proof that there is no greatest cardinality. (The uncountability of the real line easily follows, even if Cantor does not mention it because he has bigger fish to fry.) $\endgroup$ May 31, 2010 at 5:12
  • 2
    $\begingroup$ Just to fill in some history here: if I remember right, Cantor first proved the uncountability of the reals by other arguments, then later (as you reference) found the diagonal argument, as a proof of the more general statement about power sets. $\endgroup$ Sep 27, 2010 at 3:01
  • 2
    $\begingroup$ The link in the answer goes to the wrong page - it should go to page 75, not page 72. $\endgroup$
    – David Roberts
    Jun 13, 2012 at 6:41
  • 1
    $\begingroup$ And it looks like a diagonal argument to me. $\endgroup$
    – David Roberts
    Jun 13, 2012 at 6:43
7
$\begingroup$

For a bounded subset of a metric space the diameter is two times the radius!

Let $S\subset X$ be bounded. The definitions are:

$\mathrm{diameter}(S):=\sup\{d(x,y)\,|\,x,y\in S\}$

$\mathrm{radius}(S):=\inf\{r>0\,|\,\exists x\in X:\,S\subset B(x,r)\}$

where $B(x,r)$ denotes the open ball of radius $r$ around $x$.

$\endgroup$
4
  • 4
    $\begingroup$ Hםw do you define the radius of an arbitrary bounded subset? $\endgroup$
    – Mark
    Apr 11, 2011 at 15:34
  • 1
    $\begingroup$ Disproved nicely by Reuleaux triangles. $\endgroup$ Apr 12, 2011 at 8:10
  • 20
    $\begingroup$ Disproved nicely by a two-point metric space. $\endgroup$ Apr 17, 2011 at 1:36
  • 2
    $\begingroup$ An equilateral triangle in the Euclidean plane also does the job (diameter $1$ and radius $1/\sqrt{3}$): $2/\sqrt{3} > 1$. $\endgroup$ Nov 6, 2013 at 15:07
7
$\begingroup$

As a sequel of this famous answer on $\dim(U+V+W)$, the following inequality is not true $\forall n \ge 4$:
$$ \dim(\sum_{i = 1}^{n} U_i) \le \sum_{r=1}^{n} (-1)^{r+1} \sum_{i_1 < i_2 < \dots < i_r} \dim(\bigcap_{s=1}^{r}U_{i_s}) = \alpha$$
Darij Grinberg has found a counter-example (see this post).

Same flavor: for $n \le 5$, it is true that $\alpha \ge 0$ (see this proof), but it's false for $n>5$ (see this comment).

$\endgroup$
7
$\begingroup$

This may not count as a false belief, but it is an amusing misconception I had. In college I had a numerical analysis professor who had both a strong accent and messy handwriting, so it was hard to know exactly what he was talking about sometimes.

I was not yet familiar with the Greek letter $\xi$, and that was the variable he always used to represent the error of a computation, but with his handwriting it just looked like a purposeful scribble. So he would say,"Here we have the calculated value and then of course with some error" (scribble).

I thought he was just being dismissive about the error and trying to represent it in a pejorative way.

To be fair, the letter $\xi$ is not one of the easier ones to draw by hand.

$\endgroup$
2
  • 6
    $\begingroup$ One of my high school teachers was known for exclaiming "I'm a genius!" in reference to certain multivariate polynomials. $\endgroup$ Jan 13, 2018 at 17:46
  • 8
    $\begingroup$ (Well, I spent way too long trying to figure this out. In case anyone else comes across this, to save you some time, the joke is that the high school teacher was discussing homogeneous (multivariate) polynomials. With appropriate pronunciation, such as an Australian accent, "homogeneous" could sound like "I'm a genius.") $\endgroup$ Dec 14, 2018 at 9:24
7
$\begingroup$

(Cauchy and/or ordinary) product of two summable families. Until recently, I thought that, in a topological ring (i.e. a ring $R$ with topology $\tau$ such that, the maps $x\mapsto -x;\ (x,y)\mapsto x+y;\ (x,y)\mapsto x.y$ are continuous), products of two summable families were summable. In the following contexts, were my (false) beliefs

  • $(a_i)_{i\in I},\ (b_j)_{j\in J}$ supposed summable and then $(a_ib_j)_{(i,j)\in I\times J}$ is summable
  • Same situation with $I=J=\mathbb{N}$ and $c_n=\sum_{p+q=n}a_pb_q$ (Cauchy product).

But, I found this question and discussion (which proved me that this belief was false in general), returned to Bourbaki General Topology Chapter III, § 6, and there were Exercises 4-5 which proved me that this question was very delicate. Then I could debunk it.

Late addition: See also discussions and the beautiful answer by Robert Furber here.

$\endgroup$
7
$\begingroup$

Uncorrelatedness implies independence

This statement is indeed false. Suppose $X \sim U[-1, 1]$. Then $Cov(X, X^2) = EX^3 - EXEX^2 = 0$, but $X$ and $X^2$ are clearly not independent. However, that mistake is quite popular...

$\endgroup$
7
$\begingroup$

"A closed unit-ball (in a Banach space) is compact!"

It only true in finite-dimensional spaces.

$\endgroup$
2
  • 6
    $\begingroup$ Is this belief common? Alaoglu's theorem is pretty famous, and I would expect it to alert most people, even students, to the fact that the compactness of a closed unit ball is a subtle matter. $\endgroup$ Apr 19, 2022 at 22:05
  • 4
    $\begingroup$ It would be common only for complete noobs to functional analysis. $\endgroup$
    – Todd Trimble
    Apr 20, 2022 at 21:18
7
$\begingroup$

True: Given a graded algebra $A$, there is a notion of a "homogeneous" ideal of $A$. It is a property that connects an ideal of $I$ with the grading and is often necessary to require. For example, if $I$ is a homogeneous ideal of $A$, then the algebra $A / I$ is graded again. If $I$ is not homogeneous, then it is not graded in general (since the projections of different graded components of $A$ onto $A / I$ might have nonzero intersection).

False: Given a filtered algebra $A$, there is a notion of a "filtered" ideal of $A$.

There is no such notion. We can require $I$ to be generated by $I\cap A_n$ for some $n$, or actually to lie inside $A_n$ for some $n$, but in most cases none of these is actually needed. (Correct me if I am wrong.) Formulations like "Let $I$ be an ideal compatible with (or respecting) the filtration" are cargo cult.

But: Given a filtered algebra $A$ and a generating set $G$ of an ideal $I$ of $A$, it is an important question whether $I\cap A_n$ is generated by $G\cap A_n$ for every $n\in \mathbb N$. This is not always satisfied, often nontrivial (in many cases it can be proved by using the diamond lemma to show that every element of $A_n$ has a unique "remainder" modulo $I$ in a certain sense, and this remainder can be obtained by repeated subtraction multiples of elements of $G\cap A_n$) and used tacitly in various texts.

$\endgroup$
2
  • $\begingroup$ Good point, but "cargo cult"? $\endgroup$ Mar 15, 2011 at 14:32
  • 4
    $\begingroup$ What I mean is: People use these formulations as a protective charm against a danger they don't see but intuitively feel is there, although closer inspection shows that it is pure superstition. $\endgroup$ Mar 15, 2011 at 17:26
6
$\begingroup$

Here are some various examples (I hope that some of them weren't already mentioned):
1. If a space $X$ have two different norms $\| \cdot \|_i, i=1,2$ such that $\| \cdot \|_1 \leq \| \cdot \|_2$ then the completion with respect to $\| \cdot \|_1$ is contained in the completion with respect to $\| \cdot \|_2$.
2. If $M_1,M_2$ are isomorphic modules and $N_1,N_2$ are isomorphic submodules then $M_1/N_1$ and $M_2/N_2$ are isomorphic.
3. If $A,B$ are subsets of topological spaces $X,Y$ (resp.) and $A,B$ are homeomorphic then the closures $\overline{A}$ and $\overline{B}$ are also homeomorphic.
4. The standard construction of adjoining unit to the Banach algebra $A$ yields nothing new if $A$ already was unital.
5. The phrase "a function is almost everywhere continuous" means the same as: "the function is almost everywhere equal to the continuous function".
6. Suppose you are trying to prove that some function space $F$ is complete (say that functions are defined on $X$ and real valued): you take a Cauchy sequence $\{f_n\}_n$ and prove that for each point $x \in X$ the sequence $\{f_n(x)\}_n$ is Cauchy. Then form the completeness of $\mathbb{R}$ you obtain a function $f$. The false belief is that it is now enough to show that $f$ belong to $F$.
7. If you have an ascending family $\{A_i\}_i$ then to obtain it's union $\bigcup_{i}A_i$ it is enough to take some countable subfamily
8. A convergent net $\{x_i\}_i$ in a metric space is bounded and the set $\{x_i\}_i \cup \{x\}$ is compact (where $x$ is the limit).
9. If $D$ is an open dense subset of a topological space $X$ then $card \; D= card \; X$

$\endgroup$
1
6
7
8 9 10

Not the answer you're looking for? Browse other questions tagged or ask your own question.